Please answer this question now

Please Answer This Question Now

Answers

Answer 1

Answer:

Surface Area = 85.75 ft²

Step-by-step explanation:

Surface area of pyramid = ½(Perimeter of triangular base * slant height of pyramid) + Area of triangular base

Perimeter of triangular base = sum of all sides of the triangular base = 5+5+5 = 15 ft

Slant height of pyramid = 10 ft

Area of triangular base = ½*base of triangle*height of triangle = ½*5*4.3 = 10.75 ft²

Plug in the above values:

Surface Area = ½(15*10) + 10.75

= (15*5) + 10.75

= 75 + 10.75

Surface Area = 85.75 ft²


Related Questions

Find the square root of 2601 by prime factorization (USE MULTIPLICATION METHOD TO SOLVE THE ABOVE QUESTION)

Answers

2601|3

867|3

289|17

17|17

1

[tex]\sqrt{2601}=\sqrt{3^2\cdot17^2}=3\cdot17=51[/tex]

Given f(x) = 2x - 7, complete parts (a) through (c).
A. Solve f(x)=0.
B. What do the answers to parts (a) and (b) tell you about the graph of y=f(x)

Answers

Answer:

a) x=7/2

Step-by-step explanation:

a) since f(x) is=0, plug in 0 to → f(x)=2x-7 [this f(x)]. you would get 0=2x-7. solve for x by adding 7 and dividing by 2 which you get x=7/2.

Then value of [tex]x[/tex] is 7/2

What is function?

Functions are the fundamental part of the calculus in mathematics. The functions are the special types of relations. A function in math is visualized as a rule, which gives a unique output for every input . Mapping or transformation is used to denote a function in math. These functions are usually denoted by letters. The domain is defined as the set of all the values that the function can input while it can be defined. The range is all the values that come out as the output of the function involved. Co-domain is the set of values that have the potential of coming out as outputs of a function.

given function:

[tex]f(x)[/tex]= 2[tex]x[/tex] -7

So,[tex]f(x)[/tex]= 0

2[tex]x[/tex] -7=0

2[tex]x[/tex]= 7

[tex]x[/tex]= 7/2

The graph is attached below.

Learn more about function here:

https://brainly.com/question/12431044

#SPJ2

On a coordinate plane, a piecewise function has 3 lines. The first line has an open circle at (negative 9, negative 2), continues horizontally at y = negative 2, then has an open circle at (0, negative 2). The second line has an open circle at (0, 1), continues up with a positive slope, then has an open circle at (4, 9). The third line has an open circle at (4, negative 2), continues down with a negative slope, then has an open circle at (8, negative 4).
What is the domain indicated on the graph for each

Answers

Answer:  D: x = (-9, 0) U (0, 4) U (4, 8)

Step-by-step explanation:

Line 1:      y = -2               where      -9 < x < 0

Line 2:     y = 2)x + 1        where       0 < x < 4

Line 3:    y = -(1/2)x + 6    where       4 < x < 8

Domain represents the x-values.  Since all of them are open dots, the intervals are strictly less than (<).

-9 < x < 0   and   0 < x < 4   and   4 < x < 8 is the union of these intervals

-9 < x < 0   U   0 < x < 4   U   4 < x < 8

Interval Notation: D: x = (-9, 0) U (0, 4) U (4, 8)

Answer:

1st piece:  

✔ –10 < x < 0

2nd piece:  

✔ 0 < x < 4

3rd piece:  

✔ 4 < x < 8

Step-by-step explanation:

type the correct answer in the box. use numerals instead of words. what value of x makes this equation true? x/6 - 7 = -4

Answers

[tex]\dfrac{x}{6}-7=-4\\\dfrac{x}{6}=3\\x=18[/tex]

Answer:

x = 18

Step-by-step explanation:

x/6 - 7 = -4

Add 7 to each side

x/6 - 7+7 = -4+7

x/6 = 3

Multiply each side by 6

x/6 *6 = 3*6

x = 18

The numbers in the select boxes are 4 7 and 3 I put them with the photos. Can someone help

Answers

Answer:

4:7

Step-by-step explanation:

4 grape candies : 7 total candies (grape + cherry)

Answer:

It is simply 4:3

How to do this question plz answer me step by step plzz plz ​

Answers

Answer:

£13496.80

Step-by-step explanation:

We can ignore the £ sign for now, that is just units.

If we decrease a number by 4.5%, we will have to find [tex]100-4.5=95.5[/tex]% of 14132.77.

We can easily do this by setting up a proportion.

[tex]\frac{x}{14132.77} = \frac{95.5}{100}[/tex]

Multiply 14132.77 by 95.5:

[tex]14132.77\cdot95.5=1349679.535[/tex]

Divide by 100:

[tex]1349679.535\div100=13496.79535[/tex]

Rounding this to two decimal places, it simplifies to 13496.80.

Hope this helped!

Will Give Brainliest, Answer ASAP m∠O =
m∠N =

Answers

Answer:

∠ O = 61°, ∠ N = 119°

Step-by-step explanation:

In a parallelogram

Consecutive angles are supplementary

Opposite angles are congruent, thus

x + 2x - 3 = 180

3x - 3 = 180 ( add 3 to both sides )

3x = 183 ( divide both sides by 3 )

x = 61°

Thus

∠ O = ∠ M = x = 61°

∠ N = ∠ P = 2x - 3 = 2(61) - 3 = 122 - 3 = 119°

Dawn and Jackson have baseball cards in the ratio of 2:3. Together, they have a total of 60 baseball cards. How many baseball cards does each child have?

Answers

Answer:

24 and 36

Step-by-step explanation:

2x + 3x = 60

5x = 60

x = 12

Dawn has 2(12) = 24

Jackson has 3(12) = 36

Step-by-step explanation:

To find the number of baseball cards each person received we must first find the total parts

That's

2 + 3 = 5

For Dawn

Dawn's part is 2

We have

2/5 × 60

= 24 baseball cards

For Jackson

Jackson's part is 3

That's

3/5 × 60

= 36 baseball cards

Hope this helps you

The amounts of time per workout an athlete uses a stairclimber are normally​ distributed, with a mean of minutes and a standard deviation of minutes. Find the probability that a randomly selected athlete uses a stairclimber for​ (a) less than ​minutes, (b) between and ​minutes, and​ (c) more than minutes. ​(a) The probability that a randomly selected athlete uses a stairclimber for less than minutes is nothing. ​(Round to four decimal places as​ needed.) ​(b) The probability that a randomly selected athlete uses a stairclimber between and minutes is nothing. ​(Round to four decimal places as​ needed.) ​(c) The probability that a randomly selected athlete uses a stairclimber for more than minutes is nothing.

Answers

Answer:

Step-by-step explanation:

Let S be the sample space, n(S) = 60

a) Let A be the event that the selected athlete uses

s less than a minute, n(A) = 59

The probability that a randomly selected athlete uses less a minute,  P(A) = n(A)/n(S) = 59/60 = 0.9833

b) 1 - 0.9833 = 0.0167

c)  1 - 1 = 0

what is the common number to be subtracted from each term of the ratio 11:8 to get the new ratio 2:1.?​

Answers

Answer:

5 should be subtracted  from each term

Step-by-step explanation:

[tex]\frac{11-x}{8-x}=\frac{2}{1}[/tex]

Cross multiply,

1 * (11 - x) = 2*(8-x)

11 -x = 2*8 - 2*x

11 - x = 16 - 2x

Subtract 11 from both sides,

-x = 16 - 2x - 11

-x = 5 - 2x

Add 2x to both sides

-x +2x = 5 - 2 + 2x

x = 5

Julio wants to solve the system shown using the elimination method. Which is the best way to begin?
(x - 12y = 2
-4x + 7y = 12
Add the equations
b. Multiply each term in x - 12y = 2 by 4 and add it to the other original equation.
This system of equations has no solution, so Julio should not do anything.
d. Multiply each term in x - 12y = 2 by 4 and add it to the other original equation.
c.

Answers

Answer:

B. Multiply each term in x - 12y = 2 by 4 and add it to the other original equation.

Step-by-step explanation:

The expression are two linear equation and can be solved simultaneously

[tex]x - 12y = 2------------------1[/tex]

[tex]-4x + 7y = 12----------------------2[/tex]

1. we need to multiply each term in eqn 1 by 4 and add it to the other

original equation(2).

[tex]4x - 48y = 8----------------3\\-4x + 7y = 12---------------2\\\\[/tex]

Adding both 3 and 2 we have

[tex]4x - 48y = 8----------------3\\-4x + 7y = 12---------------2\\\\\\[/tex]

2. once we have gotten the value of y

we then substitute it in any of the equations to solve for x

PLS HELP I REALLY NEED IT

Answers

Answer:

x=9

Step-by-step explanation:

<B = <E  from the concurrency statement

5x = 45

Divide by 5

5x/5 = 45/5

x = 9

Answer:

Hey there!

These are similar triangles, and similar triangles have congruent angles.

Thus, we have 5x=45

Simplifying, we have x=9

Let me know if this helps :)

Brian invested his savings in two investment funds. The $8000 that he invested in Fund A returned a 4% profit. The amount that he invested in Fund B returned a 1% profit. How much did he invest in Fund B, if both funds together returned a 2% profit?

Answers

Answer: Brian invested $16000 in Fund B .

Step-by-step explanation:

Let x be the amount Brian invested in Fund B.

Given, The $8000 that he invested in Fund A returned a 4% profit. The amount that he invested in Fund B returned a 1% profit.

i.e. profit on Fund A = 4% of 8000 = 0.04 ×8000 = $320

Profit on Fund B = 1% of x = 0.01x

Together they earn 1% profit, i.e. Combined profit = 2% of (8000+x)

= 0.02(8000+x)

As per question,

Combined profit=Profit on Fund A+Profit on Fund B

[tex]\Rightarrow\ 0.02(8000+x) =320+0.01x\\\\\Rightarrow\ 0.02(8000) +0.02x=320+0.01x\\\\\Rightarrow\ 160+0.02x=320+0.01x\\\\\Rightarrow\ 0.02x-0.01x=320-160\\\\\Rightarrow\ 0.01x=160\\\\\Rightarrow\ x=\dfrac{160}{0.01}\\\\\Rightarrow\ x=16000[/tex]

Hence, Brian invested $16000 in Fund B .

WILL MARK BRAINLIEST FOR CORRECT ANSWER. <3 <3 The area of the following rectangle is 24 square units. A. Write an equation that can be used to find the value of n. B. Solve the equation to find the value of n. In your answer, show all of your work.

Answers

Because area is multiplication lets multiply the sides and make an equation:

A 2(n-3) = 24

n-3=(12)

B n= 15

15-3 = 12

12 x 2 = 24

Answer:

2(n-3)=24

n=15

Step-by-step explanation:

2(n-3)=24

2n-6=24

2n=30

n=15


what is meaning of rate?

Answers

Answer:

1.a measure, quantity, or frequency, typically one measured against another quantity or measure.

Step-by-step explanation:

2.a fixed price paid or charged for something.

Answer:

a measure, quantity, or frequency typically one measured against another quantity or measure.

Assign a standard or value to (something) according to a particular scale.

PLEASE HELP ME WORTH 20 POINTS It looks like the graph of the parents function f(x)x^2. However:
- It has been reflected (flipped) over the x-axis
-It has been shifted down 4 units.
-It had been shifted left 1 unit

Step 1: Start with the equation f(x) = x2. Write the equation for the graph of g(x) that has been reflected, or flipped, over the x-axis.

Step 2: Use the equation you wrote in Step 1. Write the equation for the graph of g(x) that has also been shifted down 4 units.

Step 3: Use the equation you wrote in Step 2. Write the equation for the graph of g(x) that has also been shifted left 1 unit.

Answers

flipped : [tex]-x^2[/tex]

moving down: [tex] -x^2+4[/tex]

shifting left [tex] -(x+1)^2+4[/tex]

expanding it: [tex] -x^2-2x+3[/tex]

Answer:

1. f(x)=x^2

f(x)=-x^2

2. f(x)=-x^2-4

3. f(x)=-(x+1)^2-4

PLS HELP. i really need this fast ill give brainliest too

Answers

Answer:

24 square units

Step-by-step explanation:

Use the formula for area of a parallelogram to solve.  The base is 6 units, and the height is 4 units.

A = bh

A = (6)(4)

A = 24 square units

The area of the parallelogram is 24 square units.

On a coordinate plane, a triangle has points (negative 5, 1), (2, 1), (2, negative 1).
Use the drop downs to answer the following questions about the distance between the points (−5, 1) and (2, −1).

What is the distance of the horizontal leg?

What is the distance of the vertical leg?

Use the Pythagorean theorem. What is the distance between the two points?

Answers

Answer:

The answer is below

Step-by-step explanation:

The points of the triangle are  (- 5, 1), (2, 1), (2, - 1). The distance between two points is given by:

[tex]Distance=\sqrt{(x_2-x_1)^2+(y_2-y_1)^2}[/tex]

The horizontal leg is formed by points with the same y axis. Therefore the points that make up the horizontal leg is (- 5, 1), (2, 1). The Distance of the horizontal leg is:

[tex]Horizontal\ leg=\sqrt{(2-(-5))^2+(1-1)^2}=\sqrt{7^2+0}=7\ units[/tex]

The vertical leg is formed by points with the same x axis. Therefore the points that make up the vertical leg is (2 1), (2, 1-). The Distance of the vertical leg is:

[tex]Vertical\ leg=\sqrt{(2-2)^2+(-1-1)^2}=\sqrt{0+(-2)^2}=2\ units[/tex]

The hypotenuse is gotten using Pythagorean theorem. It is gotten by:

Hypotenuse² = (Horizontal leg)² + (Vertical leg)²

Hypotenuse² = 7² + 2²

Hypotenuse² = 49 + 4 = 53

Hypotenuse = √53

Hypotenuse = 7.28 unit

Answer:

The answer are 7, 2 and 53

Step-by-step explanation:

Let u = , v = . Find u + v. (1 point)

Answers

Answer:

i couldnt find a solution to the equation all i could get is u +v

Step-by-step explanation:

what were you trying to say when you said "let u="

Answer:

4.8

Step-by-step explanation:

(-3,4) + (8,2) = 4.8

can u help me with this?​

Answers

Answer:  Yes. The sales tax is 5% which equals $4.20 for $84

Step-by-step explanation:

[tex]\dfrac{0.60}{12}=0.05\qquad \rightarrow 5\%\\\\\\\dfrac{1.20}{24}=0.05\qquad \rightarrow 5\%\\\\\\\dfrac{1.80}{36}=0.05\qquad \rightarrow 5\%\\\\\\\dfrac{2.40}{48}=0.05\qquad \rightarrow 5\%[/tex]

The sales tax rate is proportional for the values in the table.

$84 x 0.05 = $4.20

The sales tax on a purchase of $84 is $4.20

Puzzle corner
Look Before You Leap!
See how long it takes you to work out the
following:
(1 x2)×(3 x 4)×(586)×(7 x 8) x (
9×0)

Answers

Answer:

0

Step-by-step explanation:

Notice that the last factor is null (9×0)

So the result will be null since any number that is multiplied by 0 equals 0.

draw the graph of linear equation 5y = 3x + 18 on a cartesian plane. From the graph check weather (-2,4) is the solution of the linear equation or not PLS URGENT ANSWER

Answers

Answer:

The point (-2, 4) is not a solution of the linear equation, 5·y =  3·x + 18

Please find attached the required graph of the linear equation 5·y =  3·x + 18 written in the form y = 3/5·x + 18/5

Step-by-step explanation:

The given equation is 5·y = 3·x + 18, from which we have;

y = 3/5·x + 18/5

To draw the graph, we generate for vales of y corresponding to values of x as follows;

x,      y

-6,    0

-5,    0.6

-4,    1.2

-3,    1.8

-2,    2.4

-1,      3

0,      3.6

1,       4.2

2,      4.8

3,      5.4

4,       6

5,       6.6

6,       7.2

7,        7.8

8,        8.4

9,        9

10,      9.6

11,       10.2

12,      10.8

13,      11.4

14,       12

15,      12.6

16,      13.2

Therefore, when y = 0, x = -6, when x = 0, y = 3.6, when x = -2, y = 2.4, when y = 4, x = -2, x = 6

Therefore, the point (-2, 4) is not a solution of the linear equation, 5·y =  3·x + 18

The perimeter of an isosceles triangle is 32 inches. If the base is longer than half of the two other equal sides by 2 inches, find the length of all sides of this triangle.

Write as a equation.​

Answers

Answer:

Step-by-step explanation:

Let equal sides of an isosceles triangle = a inches

Base = [tex]\frac{1}{2}a+2[/tex] inches

Perimeter  = 32 inches

a + a + [tex]\frac{1}{2}a+2[/tex] = 32

[tex]2a + \frac{1}{2}a+2 = 32\\\\\frac{2a*2}{1*2}+\frac{1}{2}a+2=32\\\\\frac{4a}{2}+\frac{1}{2}a+2=32\\\\\frac{5}{2}a+2 = 32\\\\[/tex]

Subtract 2 from both sides

[tex]\frac{5}{2}a=32-2\\\\\frac{5}{2}a=30\\\\a=30*\frac{2}{5}\\\\a=6*2[/tex]

a = 12 inches

base = [tex]\frac{1}{2}*12+2[/tex]

        = 6 + 2

Base = 8 inches

how many cars the baseball team needs to wash before it starts making a profit. The team spent $75 setting up the car wash, and they are charging $5 per car for a wa The first step in modeling this situation is to track how much money the baseball team will take in. Write an equation to represent the amount of money collected in dollars, y, in terms of the number of cars washed, x. Ignore the setup cost.

Answers

Answer:

  y = 5x

Step-by-step explanation:

The revenue (y) is 5 dollars for each car washed. The number of cars washed is x, so the revenue equation is ...

  y = 5x

_____

Additional comment

At the end of the exercise of writing revenue and cost and profit equations, you will find that the break-even number of cars is the ratio of fixed cost (start-up cost in this case) to the profit contribution of each car (per-car charge in this case). That is, it will take 75/5 = 15 cars to break even. Each additional car will contribute a positive profit.

Answer:

Equation INCLUDING the setup cost:   y = 5x - 75

Equation EXCLUDING the setup cost:   y = 5x

Step-by-step explanation:

It spent a total of $75 to set up the car wash.

It is charging $5 per car.

y = amount collected in $

x = number of cars washed

=> We can make an equation INCLUDING the setup cost and EXCLUDING the setup cost.

=> INCLUDING the setup cost.

=> y = 5x - 75

=> I subtracted 75 from 5x because they spent a total of $75 to set up the car wash.

=> I wrote 5x because they get $5 for each car so if they wash 10 cars they get 5 * 10 = $50.

An EXAMPLE  from the above equation:

y = 5x -75

=> y = 5*14 - 75

=> y = 70 - 75

=> y = -5

=> This means that if they wash 14 cars, they still have a debt of 5 dollars.

An equation EXCLUDING the setup cost will look like:

=> y = 5x

I wrote this because, they didn't spend any money so they will get 5 dollars per car. How many cars they wash, the answer will be 'number of cars x 5'.

An EXAMPLE from the above equation is:

=> y = 5x

=> y = 5 * 14

=> y = $70

=> This means that if they wash 14 cars, they get $70.

761.8 x 10^-8 Express the number in scientific notation. A) 7.618 x 10^-6 B) 7.618 x 10^-8 C) 7.618 x 10^2 D) 7.618 x 10^6

Answers

Answer:

[tex]\huge\boxed{A)\ 7.618\times10^{-6}}[/tex]

Step-by-step explanation:

The scientific notation:

[tex]a\cdot10^n[/tex]

where

[tex]1\leq a<10;\ n\in\mathbb{Z}[/tex]

We have

[tex]761.8\times10^{-8}[/tex]

We need to move the decimal point two places to the left.

[tex]\underbrace{(7.618\times10^2)}_{=761.8}\times10^{-8}=7.618\times(10^2\times10^{-8})[/tex]

use

[tex]a^n\cdot a^m=a^{n+m}[/tex]

[tex]=7.618\times10^{2+(-8)}=7.618\times10^{-6}[/tex]

Answer:

a

Step-by-step explanation:

88 feet/second = 60 miles/hour. How many feet per second is 1 mile/hour?

Answers

Answer:

1.46666666667 feet per second

Step-by-step explanation:

60 miles per hour = 88 feet per second

=> 60/60 miles per hour = 88/60 feet per second

=> 1 mile per hour = 1.46666666667 feet per second

This bag has an unknown number of lollipops and jelly snakes in it. I know that Pr (jelly snake) = 4/5. How many jelly snakes could I have in the bag, if I know I have over 10 lollies total?

Answers

Answer:

40.

Step-by-step explanation:

Let J represent jelly snake.

Let L represent lollipops

Let S represent sample space

Let y represent the number of jelly snake in the bag. I.e

Number of J = y

Number of L = 10

Number of S = 10 + y

From the question given, we were told that the probability of jelly snake P(J) is 4/5.

But probability of jelly snake, P(J) is given by:

P(J) = nJ/nS

nJ = y

nS = 10 + y

P(J) = 4/5

Thu, we can obtain the number jelly snake, y, in the bag as follow:

P(J) = nJ/nS

4/5 = y/(10 + y)

Cross multiply

5y = 4(10 + y)

Clear bracket

5y = 40 + 4y

Collect like terms

5y – 4y = 40

y = 40

Therefore, the number of jelly snake in the bag is 40

Find the mode for the data set 18 24 24 24 25 37 37 46

Answers

Answer:

the mode would be 24

Step-by-step explanation:

it is what numbers appears most often

The mode is the number that appears most often which is 24

Maria has eight black marbles, fourteen clear marbles, and twelve blue marbles in a bag. If she picks two marbles at random, without replacement, what is the probability that she will select a blue marble first, then a clear marble?

Answers

Answer:

[tex]\boxed{0.15}[/tex]

Step-by-step explanation:

Part 1: Solve for the total amount of marbles

To solve for the probability of certain events, a population is needed to derive this information from. In order to find this population, add up the amounts of each marble.

8 + 14 + 12 = 34 marbles

Part 2: Determine the probabilities

Now, given the amounts of marbles, simply multiply the ratios of blue marbles to total marbles and the ratio of clear marbles to total marbles to get the combined probability.

[tex]\frac{12}{34}*\frac{14}{33} = \frac{28}{187} \approxeq 0.1497 \approxeq 0.15 * 100 = 15[/tex]

The probability of these events occurring simultaneously is 15%.

Which number line represents the solution set for the inequality –negative StartFraction one-half EndFraction x is greater than or equal to 4.x ≥ 4?

A number line from negative 10 to 10 in increments of 2. A point is at negative 2 and a bold line starts at negative 2 and is pointing to the left.
A number line from negative 10 to 10 in increments of 2. A point is at negative 8 and a bold line starts at negative 8 and is pointing to the left.
A number line from negative 10 to 10 in increments of 2. A point is at negative 2 and a bold line starts at negative 2 and is pointing to the right.
A number line from negative 10 to 10 in increments of 2. A point is at negative 8 and a bold line starts at negative 8 and is pointing to the right.

Answers

Answer:

it's b :)

Step-by-step explanation:

A number line which represents the solution set for the given inequality is: option B.

What is a number line?

A number line refers to a type of graph with a graduated straight line which contains numerical values (both positive and negative numbers) that are placed at equal intervals along its length.

Next, we would solve the given inequality:

-½x ≥ 4

-x ≥ 4 × 2

x ≤ -8.

Therefore, a number line which represents the solution set for the given inequality is a number line from -10 to 10 in increments of 2 with a point at -8 and a bold line starts at -8 while pointing to the left.

Read more on number line here: brainly.com/question/24644930

#SPJ9

Other Questions
Why did southern planters switch from using indentured servants to using slaves? SELECT ALL THAT APPLY 1) Slaves could be controlled more easily than other workers.2) Indentured servants were charging high fees for their labor. 3) Many former indentured servants had participated in a rebellion. 4) Slaves could do other jobs like shoeing horses or cooking. Which point lies on the line with point-slope equation y - 3 = 4(x + 7)?A.(7, 3)B.(7, -3)C.(-7, -3)D.(-7, 3) PLZ HELP THANKS! Find the equation of the line passing through the pair points (-8,6) (-9,-9). The equation of the line in the form is Ax+By=C. Your customer purchases FYZ 4% convertible preferred stock at $60 per share. The conversion price is $10. With the common stock now trading 1 point below parity, you know that the common shares of FYZ are trading at In this activity, you will conduct online research to collect information about the evolution of the AFN industry in the United States. Pick one area that interests you and explore that topic and write a brief report on the major agricultural inventions and their impact on human life. PLEASE HELP WITH THIS OR GIVE ME AN IDEA OF WHAT TO WRITE!! YOU DON'T HAVE TO GIVE A FULL REPORT!! A box contains 20 equal-sized balls, numbered 1 to 20. Two balls are drawn at random simultaneously. What is the probability that the numbers on the two balls will differ by more than 2 Find the mean, median, and mode Light of wavelength 520 nm is incident a on a diffraction grating with a slit spacing of 2.20 m , what is the angle from the axis for the third order maximum? An 85-year old risk averse investor is not happy about the minimal return she is earning on her current investments. She is stressed about having enough income because her cost of living has been increasing by more than 10% annually. Her current portfolio composition consists of: 40% Money Market Fund 50% Bonds 10% Equities What changes should you suggest to her portfolio? A. Reduce the Money Market Fund allocation by 10% (to 30%) and put the released funds in commodities such as gold B. Reduce the Money Market Fund allocation by 30% (to 10%) and put the released funds in AAA-rated corporate bonds C. Liquidate the entire Money Market Fund allocation and put the released funds in Equities, bringing that allocation up to 50% D. Liquidate the entire Money Market Fund allocation and put the released funds in U.S. Treasury securities Read the excerpt from The Code Book. A theoretical breakthrough would be a fundamentally new way of finding Alice's private key. Alice's private key consists of p and q, and these are found by factoring the public key, N. The standard approach is to check each prime number one at a time to see if it divides into N, but we know that this takes an unreasonable amount of time. Which statement is best supported by this excerpt Which words could correctly replace the underlined words in the sentence below?Today was an exhausting day: I can't wait to relax this weekend.a tranquila peacefula gruelinga serene In a reversible reaction, the endothermic reaction absorbs ____________ the exothermic reaction releases.A. None of these, endothermic reactions release energyB. more energy thanC. the same amount of energy asD. less energy than Who supported the Nationalist Party in the Chinese Civil War? Germany the USSR the United States the rest of Asia The content of total body water in in adult women is * Describe the tone Grandma uses when she speaks. What kind of words help to create this tone? In a monster calls Test the claim that the proportion of people who own cats is significantly different than 80% at the 0.2 significance level. The null and alternative hypothesis would be:______. A. H0 : = 0.8 H 1 : 0.8 B. H0 : p 0.8 H 1 : p > 0.8 C. H0 : p = 0.8 H 1 : p 0.8 D. H0 : 0.8 H 1 : > 0.8 E. H0 : p 0.8 H 1 : p < 0.8 F. H0 : 0.8 H 1 : < 0.8 The test is:_____. a. left-tailed b. right-tailed c. two-tailed Based on a sample of 200 people, 79% owned cats.The test statistic is:______.The p-value is:_____. Based on this we:_____.A. Fail to reject the null hypothesis.B. Reject the null hypothesis. In three years, 30% of a radioactive element decays. Find its half-life. (Round your answer to one decimal place.) yr greg suffers from depression. His friend matthew also suffers from depression as well as conduct disorder. According to the theory of problem behavior syndrome, this means A 2-gallon bottle of fabric softener costs $30.24. What is the price per pint? Oligopoly firms will seldom change prices but if one firm increases their price, others may follow if costs have ____________ .